www.matheraum.de
Das Matheforum.
Das Matheforum des MatheRaum.

Für Schüler, Studenten, Lehrer, Mathematik-Interessierte.
Hallo Gast!einloggen | registrieren ]
Startseite · Forum · Wissen · Kurse · Mitglieder · Team · Impressum
Forenbaum
^ Forenbaum
Status Mathe
  Status Schulmathe
    Status Primarstufe
    Status Mathe Klassen 5-7
    Status Mathe Klassen 8-10
    Status Oberstufenmathe
    Status Mathe-Wettbewerbe
    Status Sonstiges
  Status Hochschulmathe
    Status Uni-Analysis
    Status Uni-Lin. Algebra
    Status Algebra+Zahlentheo.
    Status Diskrete Mathematik
    Status Fachdidaktik
    Status Finanz+Versicherung
    Status Logik+Mengenlehre
    Status Numerik
    Status Uni-Stochastik
    Status Topologie+Geometrie
    Status Uni-Sonstiges
  Status Mathe-Vorkurse
    Status Organisatorisches
    Status Schule
    Status Universität
  Status Mathe-Software
    Status Derive
    Status DynaGeo
    Status FunkyPlot
    Status GeoGebra
    Status LaTeX
    Status Maple
    Status MathCad
    Status Mathematica
    Status Matlab
    Status Maxima
    Status MuPad
    Status Taschenrechner

Gezeigt werden alle Foren bis zur Tiefe 2

Navigation
 Startseite...
 Neuerdings beta neu
 Forum...
 vorwissen...
 vorkurse...
 Werkzeuge...
 Nachhilfevermittlung beta...
 Online-Spiele beta
 Suchen
 Verein...
 Impressum
Das Projekt
Server und Internetanbindung werden durch Spenden finanziert.
Organisiert wird das Projekt von unserem Koordinatorenteam.
Hunderte Mitglieder helfen ehrenamtlich in unseren moderierten Foren.
Anbieter der Seite ist der gemeinnützige Verein "Vorhilfe.de e.V.".
Partnerseiten
Mathe-Seiten:Weitere Fächer:

Open Source FunktionenplotterFunkyPlot: Kostenloser und quelloffener Funktionenplotter für Linux und andere Betriebssysteme
StartseiteMatheForenUni-Komplexe AnalysisFourierreihe und Laurentreihe
Foren für weitere Schulfächer findest Du auf www.vorhilfe.de z.B. Deutsch • Englisch • Französisch • Latein • Spanisch • Russisch • Griechisch
Forum "Uni-Komplexe Analysis" - Fourierreihe und Laurentreihe
Fourierreihe und Laurentreihe < komplex < Analysis < Hochschule < Mathe < Vorhilfe
Ansicht: [ geschachtelt ] | ^ Forum "Uni-Komplexe Analysis"  | ^^ Alle Foren  | ^ Forenbaum  | Materialien

Fourierreihe und Laurentreihe: Frage (beantwortet)
Status: (Frage) beantwortet Status 
Datum: 11:55 Mi 13.03.2013
Autor: loggeli

Aufgabe
Stellen Sie die Fourierreihe von

\(f : \mathbb{R} \rightarrow \mathbb{R}\), \begin{equation} f(t) = \frac{\sin(t)}{2+\cos(t)} \end{equation}

mit Hilfe einer Laurentreihe einer geeigneten holomorphen Funktion g auf.

Hallo liebe Mathefreunde,

ich habe zunächst die Funktion in eine komplexe holomorphe Funktion umgeschrieben.

\(g(z) = i\cdot \frac{(1-z^{2})}{z^{2}+4z+1}\)

Hier habe ich jetzt eine Polynomdivision und eine Partialbruchzerlegung ausgeführt.

\(g(z) = -i + i\cdot (\sqrt{3}+2) \cdot \frac{1}{z-(-2-\sqrt{3})} - i (2-\sqrt{3}) \cdot \frac{1}{z-(-2+\sqrt{3})}\)

Diesen Term habe ich dann jeweils nach Fallunterscheidung in eine Laurentreihe entwickelt.

\begin{equation} g(z)= \begin{cases} -i + i\cdot \sum_{n=0}^{\infty} [(-2+\sqrt{3})^{-n} + (-2+\sqrt{3})^{n}]z^{n} & |z| < -2 + \sqrt{3}\\\\ -i + i\cdot \sum_{n=0}^{\infty} (-2-\sqrt{3})^{-n} z^{n}+ i\cdot \sum_{n=-\infty}^{-1} (-2+\sqrt{3})^{-n} z^{n} & -2 + \sqrt{3} < |z| < - 2 - \sqrt{3}\\\\ -i + i\cdot \sum_{n=-\infty}^{-1} (-2-\sqrt{3})^{-n} z^{n} + i\cdot \sum_{n=-\infty}^{-1} (-2+\sqrt{3})^{-n} z^{n} & |z| > -2-\sqrt{3}\\ \end{cases} \end{equation}

Meine Frage:
Nun weiß ich leider nich genau, welche der Konvergenzberreiche ich für die Entwicklung in eine Fourierreihe benutzen soll. Könnt ihr mir da vielleicht weiterhelfen?

Ich freue mich auf eine Antwort :-)

Beste Grüße,
loggeli

        
Bezug
Fourierreihe und Laurentreihe: Antwort
Status: (Antwort) fertig Status 
Datum: 16:09 Do 14.03.2013
Autor: MathePower

Hallo loggeli,

> Stellen Sie die Fourierreihe von
>
> > \(f : \mathbb{R} \rightarrow \mathbb{R}\), > \begin{equation} > f(t) = \frac{\sin(t)}{2+\cos(t)} > \end{equation} >
>  
> mit Hilfe einer Laurentreihe einer geeigneten holomorphen
> Funktion g auf.
>  Hallo liebe Mathefreunde,
>  
> ich habe zunächst die Funktion in eine komplexe holomorphe
> Funktion umgeschrieben.
>  
> > \(g(z) = i\cdot \frac{(1-z^{2})}{z^{2}+4z+1}\) >
>  
> Hier habe ich jetzt eine Polynomdivision und eine
> Partialbruchzerlegung ausgeführt.
>  
> > \(g(z) = -i + i\cdot (\sqrt{3}+2) \cdot \frac{1}{z-(-2-\sqrt{3})} - i (2-\sqrt{3}) \cdot \frac{1}{z-(-2+\sqrt{3})}\) >
>  
> Diesen Term habe ich dann jeweils nach Fallunterscheidung
> in eine Laurentreihe entwickelt.
>  
> > \begin{equation} > g(z)= > \begin{cases} > -i + i\cdot \sum_{n=0}^{\infty} [(-2+\sqrt{3})^{-n} + (-2+\sqrt{3})^{n}]z^{n} & |z| < -2 + \sqrt{3}\\\\ > -i + i\cdot \sum_{n=0}^{\infty} (-2-\sqrt{3})^{-n} z^{n}+ i\cdot \sum_{n=-\infty}^{-1} (-2+\sqrt{3})^{-n} z^{n} & -2 + \sqrt{3} < |z| < - 2 - \sqrt{3}\\\\ > -i + i\cdot \sum_{n=-\infty}^{-1} (-2-\sqrt{3})^{-n} z^{n} + i\cdot \sum_{n=-\infty}^{-1} (-2+\sqrt{3})^{-n} z^{n} & |z| > -2-\sqrt{3}\\ > \end{cases} > \end{equation} >
>  
> Meine Frage:
>  Nun weiß ich leider nich genau, welche der
> Konvergenzberreiche ich für die Entwicklung in eine
> Fourierreihe benutzen soll. Könnt ihr mir da vielleicht
> weiterhelfen?
>  


Benutze den Kreisring.

[mm]-2 + \sqrt{3} < |z| < 2 + \sqrt{3}[/mm]


> Ich freue mich auf eine Antwort :-)
>  
> Beste Grüße,
>  loggeli


Gruss
MathePower

Bezug
                
Bezug
Fourierreihe und Laurentreihe: Frage (beantwortet)
Status: (Frage) beantwortet Status 
Datum: 17:08 Do 14.03.2013
Autor: loggeli

Hallo Mathepower,

danke für die Antwort. Ich habe nun folgendermaßen weitergerechnet:
g(z) = -i + i\cdot \sum_{n=0}^{\infty} (-2-\sqrt{3})^{-n} z^{n}+ i\cdot \sum_{n=-\infty}^{-1} (-2+\sqrt{3})^{-n} z^{n}

g(z) = -i + i\cdot \sum_{n=0}^{\infty} (-2+\sqrt{3})^{n} z^{n}- i\cdot \sum_{n=0}^{\infty} (-2+\sqrt{3})^{-n} z^{-n}

g(z) = -i + i\cdot \sum_{n=0}^{\infty} (-2+\sqrt{3})^{n} (z^{n} - z^{-n})

z^{n} = cos(n\phi) + i\cdot sin(n\varphi)

g(z) = -i + i\cdot \sum_{n=0}^{\infty} (-2+\sqrt{3})^{n} (2i\sin(n\varphi))

Somit bekomme ich als Ergebnis für die Fourierreihe
g(z) = -i - 2\cdot \sum_{n=0}^{\infty} (-2+\sqrt{3})^{n} (\sin(n\varphi))

Wäre sehr nett, wenn Du mir sagen könntest, ob das so richtig ist, bzw. wo ich da nen Fehler reingehauen hab. Vielen Dank :-)

Grüße,
loggeli

Bezug
                        
Bezug
Fourierreihe und Laurentreihe: Antwort
Status: (Antwort) fertig Status 
Datum: 18:35 Do 14.03.2013
Autor: MathePower

Hallo loggeli,

> Hallo Mathepower,
>  
> danke für die Antwort. Ich habe nun folgendermaßen
> weitergerechnet:
>   > > g(z) = -i + i\cdot \sum_{n=0}^{\infty} (-2-\sqrt{3})^{-n} z^{n}+ i\cdot \sum_{n=-\infty}^{-1} (-2+\sqrt{3})^{-n} z^{n} > >
>  
> > > g(z) = -i + i\cdot \sum_{n=0}^{\infty} (-2+\sqrt{3})^{n} z^{n}- i\cdot \sum_{n=0}^{\infty} (-2+\sqrt{3})^{-n} z^{-n} > >
>  
> > > g(z) = -i + i\cdot \sum_{n=0}^{\infty} (-2+\sqrt{3})^{n} (z^{n} - z^{-n}) > >
>  
> > z^{n} = cos(n\phi) + i\cdot sin(n\varphi) >
>  
> > > g(z) = -i + i\cdot \sum_{n=0}^{\infty} (-2+\sqrt{3})^{n} (2i\sin(n\varphi)) > >
>  
> Somit bekomme ich als Ergebnis für die Fourierreihe
>   > g(z) = -i - 2\cdot \sum_{n=0}^{\infty} (-2+\sqrt{3})^{n} (\sin(n\varphi)) >
>  


Das "-i" ist zuviel, ansonsten stimmt die erhaltene Fourierreihe.


> Wäre sehr nett, wenn Du mir sagen könntest, ob das so
> richtig ist, bzw. wo ich da nen Fehler reingehauen hab.
> Vielen Dank :-)
>  
> Grüße,
>  loggeli


Gruss
MathePower

Bezug
                                
Bezug
Fourierreihe und Laurentreihe: Frage (beantwortet)
Status: (Frage) beantwortet Status 
Datum: 10:38 Fr 15.03.2013
Autor: loggeli

Hallo Mathepower,

danke für die Antwort. Ich hatte mir schon gedacht, dass das i da zu viel ist, da die Fourierreihe ja eigentlich reell sein sollte.

Das i kommt bei mir wegen der Polynomdivision von g(z) mit ins Spiel.

g(z) = -i - 2\cdot \sum_{n=0}^{\infty} (-2+\sqrt{3})^{n} \sin(n\varphi)

Für n=0 gilt ja g(z)=-i. Kann man dann für g(z) einfach dieses hier schreiben

g(z) = - 2\cdot \sum_{n=1}^{\infty} (-2+\sqrt{3})^{n} \sin(n\varphi)

Habe jetzt die Summe bei n=1 loslaufen lassen.

Grüße,
loggeli

Bezug
                                        
Bezug
Fourierreihe und Laurentreihe: Antwort
Status: (Antwort) fertig Status 
Datum: 15:53 Fr 15.03.2013
Autor: MathePower

Hallo loggeli,

> Hallo Mathepower,
>  
> danke für die Antwort. Ich hatte mir schon gedacht, dass
> das i da zu viel ist, da die Fourierreihe ja eigentlich
> reell sein sollte.
>


Für die erste Summe lautet das erste Glied "i".
Damit ergibt -i+i=0.


> Das i kommt bei mir wegen der Polynomdivision von g(z) mit
> ins Spiel.
>
> > g(z) = -i - 2\cdot \sum_{n=0}^{\infty} (-2+\sqrt{3})^{n} \sin(n\varphi) >
>  
> Für n=0 gilt ja g(z)=-i. Kann man dann für g(z) einfach
> dieses hier schreiben
>  
> > g(z) = - 2\cdot \sum_{n=1}^{\infty} (-2+\sqrt{3})^{n} \sin(n\varphi) >
>  
> Habe jetzt die Summe bei n=1 loslaufen lassen.
>  
> Grüße,
>  loggeli


Gruss
MathePower

Bezug
                                                
Bezug
Fourierreihe und Laurentreihe: Frage (beantwortet)
Status: (Frage) beantwortet Status 
Datum: 11:18 Sa 16.03.2013
Autor: loggeli

Hallo,

mir sind noch zwei letzte Fragen zu der Sache aufgekommen:

1.) Erstes Glied der Summe

$ > g(z) = -i - [mm] 2\cdot \sum_{n=0}^{\infty} (-2+\sqrt{3})^{n} \sin(n\varphi) [/mm] > $

Hier ist bei n=0 die Summe doch 0 oder? Ich steh da grad etwas auf dem Schlauch.

2.) Berreich der Laurentreihe

Nimmt man grad den mittleren Entwicklungsberreich, weil hier die Laurentreihe absolut konvergiert, oder gibt es da noch einen anderen Grund? Mir war am Anfang garnicht klar, welchen Berreich ich jetzt für die Fourierreihe nehmen sollte.

Wäre sehr nett, wenn Ihr mir dazu etwas sagen könntet :-)

Viele Grüße,
loggeli


Bezug
                                                        
Bezug
Fourierreihe und Laurentreihe: Antwort
Status: (Antwort) fertig Status 
Datum: 16:41 Sa 16.03.2013
Autor: MathePower

Hallo loggeli,

> Hallo,
>  
> mir sind noch zwei letzte Fragen zu der Sache aufgekommen:
>  
> 1.) Erstes Glied der Summe
>  
> [mm]> g(z) = -i - 2\cdot \sum_{n=0}^{\infty} (-2+\sqrt{3})^{n} \sin(n\varphi) >[/mm]
>  
> Hier ist bei n=0 die Summe doch 0 oder? Ich steh da grad
> etwas auf dem Schlauch.
>  


Ich bin von dieser Darstellung ausgegangen:

[mm] g(z) = -i + i\cdot \sum_{n=0}^{\infty} (-2-\sqrt{3})^{-n} z^{n}+ i\cdot \sum_{n=-\infty}^{-1} (-2+\sqrt{3})^{-n} z^{n}[/mm]

Die erste Summe ist

[mm]i\cdot \sum_{n=0}^{\infty} (-2-\sqrt{3})^{-n} z^{n}[/mm]

Davon das erste Glied ist: [mm]i\cdot (-2-\sqrt{3})^{-0} z^{0}=i[/mm]


> 2.) Berreich der Laurentreihe
>  
> Nimmt man grad den mittleren Entwicklungsberreich, weil
> hier die Laurentreihe absolut konvergiert, oder gibt es da
> noch einen anderen Grund? Mir war am Anfang garnicht klar,
> welchen Berreich ich jetzt für die Fourierreihe nehmen
> sollte.

>


Ein möglicher Grund ist doch offensichtlich:

Es tauchen positive wie negative Exponenten auf.

Im Falle der beiden anderen Entwicklungsbereiche
tauchen entweder nur positive oder negative Exponenten auf.


> Wäre sehr nett, wenn Ihr mir dazu etwas sagen könntet
> :-)
>  
> Viele Grüße,
>  loggeli

>


Gruss
MathePower  

Bezug
Ansicht: [ geschachtelt ] | ^ Forum "Uni-Komplexe Analysis"  | ^^ Alle Foren  | ^ Forenbaum  | Materialien


^ Seitenanfang ^
www.matheforum.net
[ Startseite | Forum | Wissen | Kurse | Mitglieder | Team | Impressum ]